% theme: Programme de calculs %- set a = randint(5, 15) %- set b = randint(2, 15) %- set c = b - 3 %- set d = a * b Voici un programme de calcul. \begin{center} \fbox{\colorbox{base2}{ \begin{minipage}[h]{0.5\textwidth} Prendre un nombre\\ Lui ajouter \Var{a} \\ Multiplier le résultat par \Var{b}\\ Enlever \Var{c} fois le nombre de départ\\ Enlever \Var{d} \end{minipage} }} \end{center} \begin{enumerate} %- set choix1 = randint(5, 20) %- set ans1 = choix1 * 3 \item Vérifier quand quand on choisit \Var{choix1} on obtient \Var{ans1} à la fin. \begin{solution} \begin{eqnarray*} \Var{choix1} \rightarrow %- set e1 = choix1 + a \Var{e1} \rightarrow %- set e2 = e1 * a \Var{e2} \rightarrow %- set e3 = e2 - choix1 * c \Var{e3} \rightarrow %- set e4 = e2 - d \Var{e4} \end{eqnarray*} \end{solution} %- set choix2 = randint(5, 20) %- set ans2 = choix2 * 3 %- if choix1 == choix2 %- set choix2 = -choix1 %- set ans2 = choix2 * 3 %- endif \item Quel nombre obtient-on quand on choisit \Var{choix2}? \begin{solution} \begin{eqnarray*} \Var{choix2} \rightarrow %- set e1 = choix1 + a \Var{e1} \rightarrow %- set e2 = e1 * b \Var{e2} \rightarrow %- set e3 = e2 - choix1 * c \Var{e3} \rightarrow %- set e4 = e2 - d \Var{e4} \end{eqnarray*} \end{solution} \item Démontrer que quand on applique le programme à $x$, on obtient \begin{eqnarray*} \Var{b}(x + \Var{a}) - \Var{c}x - \Var{d} \end{eqnarray*} %- set end3 = randint(20, 40) %- if not (end3 % 3) %- set end3 = end3 + 1 %- endif \item Quel nombre doit-on choisir pour obtenir \Var{end3} à la fin? \begin{solution} On doit choisir $\dfrac{\Var{end3}}{3}$ \end{solution} \item Démontrer que pour n'importe quel nombre choisi, on obtient toujours le triple de ce nombre. \end{enumerate}